免費論壇 繁體 | 簡體
Sclub交友聊天~加入聊天室當版主
分享
返回列表 发帖

[数列] 转发一个数列问题

已知正项数列{$x_n$}满足$x_n+\frac{1}{x_{n+1}}<2,n\ge1$.
1.证明:$x_n+\frac{1}{x_{n}}\ge2$;
2.证明:$x_n<x_{n+1}$;
3.证明:$\frac{n-1}{n}<x_n<\frac{n+1}{n}$.
分享到: QQ空间QQ空间 腾讯微博腾讯微博 腾讯朋友腾讯朋友

本帖最后由 tommywong 于 2014-5-21 17:45 编辑

1.$x_n>0$
2.对$x_n-2$用条件
3.
$a_n<x_n<b_n$
$a_1=0$
$\displaystyle a_n+\frac{1}{x_{n+1}}<x_n+\frac{1}{x_{n+1}}<2$
$\displaystyle x_{n+1}>\frac{1}{2-a_n}=a_{n+1}$
$\displaystyle x_{n+1}>\frac{1}{2-x_n}>0,0<x_n<2,b_n=2$
$\displaystyle x_n+\frac{1}{b_{n+1}}<x_n+\frac{1}{x_{n+1}}<2,x_n<2-\frac{1}{b_{n+1}}=b_n,b_n=1$

TOP

$\displaystyle a_{n+1}=\frac{1}{2-a_n}$

$a_n=y_n+1$

$\displaystyle y_{n+1}+1=\frac{1}{1-y_n}$

$\displaystyle y_{n+1}=\frac{y_n}{1-y_n}=\frac{1}{\frac{1}{y_n}-1},\frac{1}{y_{n+1}}=\frac{1}{y_n}-1$

$\displaystyle z_n=\frac{1}{y_n}$

$z_{n+1}=z_n-1=z_1+1-n=-n$

$\displaystyle y_n=\frac{-1}{n},a_n=\frac{n-1}{n}$

TOP

这两天精神不佳,不知这样证明有没有问题,加强了右边,请大家检查。

前两问太显然,略;

第(3)问,左边由数归易证,也略。

下面将第(3)问右边加强为 $x_n<1$。

先证明一个结论,在题设下,不存在 $p\geqslant 0$, $t>0$ 以及 $x_k$ 满足
\[x_k>\frac{1-pt}{1-(p+1)t}~\text{且}~t<\frac1{p+3},\]

用反证法,假设存在,则
\[2>x_k+\frac1{x_{k+1}}>\frac{1-pt}{1-(p+1)t}+\frac1{x_{k+1}},\]

\[\frac1{x_{k+1}}<2-\frac{1-pt}{1-(p+1)t}=\frac{1-(p+2)t}{1-(p+1)t},\]
因为 $t<1/(p+3)$,故上式两边均为正,故
\[x_{k+1}>\frac{1-(p+1)t}{1-(p+2)t},\]
又 $0<x_{k+2}<2$,故
\[2>x_{k+1}+\frac1{x_{k+2}}>x_{k+1}+\frac12\riff x_{k+1}<\frac32,\]
因此
\[\frac{1-(p+1)t}{1-(p+2)t}<\frac32,\]
解得
\[t<\frac1{p+4},\]
这样我们就得到
\[x_{k+1}>\frac{1-(p+1)t}{1-(p+2)t}~\text{且}~t<\frac1{p+4},\]
递推下去,即得对任意 $m\in\mbb N^+$,都有
\[x_{k+m}>\frac{1-(p+m)t}{1-(p+m+1)t}~\text{且}~t<\frac1{p+m+3},\]
令 $m\to\infty$,就与 $t>0$ 矛盾,结论得证。

回到证明 $x_n<1$ 上面,假设存在某个 $x_n=1+t$ 且 $t\geqslant 0$。

(1)若 $t>0$,则由
\[
2>x_n+\frac1{x_{n+1}}=1+t+\frac1{x_{n+1}} \iff
\frac1{x_{n+1}}<1-t,
\]
显然 $t<1$,故
\[x_{n+1}>\frac1{1-t},\]
又 $x_{n+1}<3/2$,故
\[\frac1{1-t}<\frac32 \riff t<\frac13,\]
而这是前面结论中 $p=0$ 的情形,所以这是不可能的;

(2)若 $t=0$,即 $x_n=1$,则显然 $x_{n+1}>1$,于是又回到(1)上面去,也不可能。

综上所述,$x_n<1$ 恒成立。
$\href{https://kuingggg.github.io/}{\text{About Me}}$

TOP

银哩?

TOP

TOP

本帖最后由 realnumber 于 2014-5-22 21:32 编辑

对任意n,由$x_n<2$得$x_{n-1}<\frac{3}{2}$,重复这一步,可得$x_{n-2}<\frac{4}{3}$,$x_{n-3}<\frac{5}{4}$,$x_{n-4}<\frac{6}{5}....$$x_{n-k}<\frac{k+2}{k+1}$,.....
即意味着对任意正整数k,任意的n,总有$a_n<\frac{k+2}{k+1}$.
这样因为数列递增,若有$x_{n_0-1}>1+t,t\ge0$,则有$x_{n_0}>1+s,0.5>s>\frac{1}{k}$,存在某个实数s,与上面一行矛盾.

TOP

回复 7# realnumber

看上去跟4#差不多,至少在想法上……

TOP

回复 8# kuing


    应该是的,就这么个已知条件.

TOP

返回列表 回复 发帖